How to prove that $frac{zeta(2) }{2}+frac{zeta (4)}{2^3}+frac{zeta (6)}{2^5}+frac{zeta (8)}{2^7}+cdots=1$?












75














How can one prove this identity?




$$frac{zeta(2) }{2}+frac{zeta (4)}{2^3}+frac{zeta (6)}{2^5}+frac{zeta (8)}{2^7}+cdots=1$$






There is a formula for $zeta$ values at even integers, but it involves Bernoulli numbers; simply plugging it in does not appear to be an efficient approach.










share|cite|improve this question




















  • 12




    I think this is a normal question. I don't know why "on hold"?
    – E.H.E
    Dec 24 '14 at 19:28






  • 2




    There is a recursive formula for $zeta(2n)$ derived in this answer. However, in that answer it is shown that $$sum_{k=1}^inftyzeta(2k)x^{2k}=frac12(1-pi xcot(pi x))$$ With $x=frac12$, this immediately gives $$sum_{k=1}^inftyzeta(2k)left(frac12right)^{2k-1} =1-fracpi2cotleft(fracpi2right)=1$$ This is what is used in Random Variable's answer.
    – robjohn
    Dec 31 '14 at 18:13


















75














How can one prove this identity?




$$frac{zeta(2) }{2}+frac{zeta (4)}{2^3}+frac{zeta (6)}{2^5}+frac{zeta (8)}{2^7}+cdots=1$$






There is a formula for $zeta$ values at even integers, but it involves Bernoulli numbers; simply plugging it in does not appear to be an efficient approach.










share|cite|improve this question




















  • 12




    I think this is a normal question. I don't know why "on hold"?
    – E.H.E
    Dec 24 '14 at 19:28






  • 2




    There is a recursive formula for $zeta(2n)$ derived in this answer. However, in that answer it is shown that $$sum_{k=1}^inftyzeta(2k)x^{2k}=frac12(1-pi xcot(pi x))$$ With $x=frac12$, this immediately gives $$sum_{k=1}^inftyzeta(2k)left(frac12right)^{2k-1} =1-fracpi2cotleft(fracpi2right)=1$$ This is what is used in Random Variable's answer.
    – robjohn
    Dec 31 '14 at 18:13
















75












75








75


38





How can one prove this identity?




$$frac{zeta(2) }{2}+frac{zeta (4)}{2^3}+frac{zeta (6)}{2^5}+frac{zeta (8)}{2^7}+cdots=1$$






There is a formula for $zeta$ values at even integers, but it involves Bernoulli numbers; simply plugging it in does not appear to be an efficient approach.










share|cite|improve this question















How can one prove this identity?




$$frac{zeta(2) }{2}+frac{zeta (4)}{2^3}+frac{zeta (6)}{2^5}+frac{zeta (8)}{2^7}+cdots=1$$






There is a formula for $zeta$ values at even integers, but it involves Bernoulli numbers; simply plugging it in does not appear to be an efficient approach.







real-analysis sequences-and-series complex-analysis zeta-functions






share|cite|improve this question















share|cite|improve this question













share|cite|improve this question




share|cite|improve this question








edited Dec 25 '14 at 1:05







user147263

















asked Dec 24 '14 at 18:24









E.H.E

15.7k11966




15.7k11966








  • 12




    I think this is a normal question. I don't know why "on hold"?
    – E.H.E
    Dec 24 '14 at 19:28






  • 2




    There is a recursive formula for $zeta(2n)$ derived in this answer. However, in that answer it is shown that $$sum_{k=1}^inftyzeta(2k)x^{2k}=frac12(1-pi xcot(pi x))$$ With $x=frac12$, this immediately gives $$sum_{k=1}^inftyzeta(2k)left(frac12right)^{2k-1} =1-fracpi2cotleft(fracpi2right)=1$$ This is what is used in Random Variable's answer.
    – robjohn
    Dec 31 '14 at 18:13
















  • 12




    I think this is a normal question. I don't know why "on hold"?
    – E.H.E
    Dec 24 '14 at 19:28






  • 2




    There is a recursive formula for $zeta(2n)$ derived in this answer. However, in that answer it is shown that $$sum_{k=1}^inftyzeta(2k)x^{2k}=frac12(1-pi xcot(pi x))$$ With $x=frac12$, this immediately gives $$sum_{k=1}^inftyzeta(2k)left(frac12right)^{2k-1} =1-fracpi2cotleft(fracpi2right)=1$$ This is what is used in Random Variable's answer.
    – robjohn
    Dec 31 '14 at 18:13










12




12




I think this is a normal question. I don't know why "on hold"?
– E.H.E
Dec 24 '14 at 19:28




I think this is a normal question. I don't know why "on hold"?
– E.H.E
Dec 24 '14 at 19:28




2




2




There is a recursive formula for $zeta(2n)$ derived in this answer. However, in that answer it is shown that $$sum_{k=1}^inftyzeta(2k)x^{2k}=frac12(1-pi xcot(pi x))$$ With $x=frac12$, this immediately gives $$sum_{k=1}^inftyzeta(2k)left(frac12right)^{2k-1} =1-fracpi2cotleft(fracpi2right)=1$$ This is what is used in Random Variable's answer.
– robjohn
Dec 31 '14 at 18:13






There is a recursive formula for $zeta(2n)$ derived in this answer. However, in that answer it is shown that $$sum_{k=1}^inftyzeta(2k)x^{2k}=frac12(1-pi xcot(pi x))$$ With $x=frac12$, this immediately gives $$sum_{k=1}^inftyzeta(2k)left(frac12right)^{2k-1} =1-fracpi2cotleft(fracpi2right)=1$$ This is what is used in Random Variable's answer.
– robjohn
Dec 31 '14 at 18:13












4 Answers
4






active

oldest

votes


















107














Since
$$zeta(2n) = frac{1}{(2n-1)!}int_{0}^{+infty}frac{x^{2n-1}}{e^x-1},dx $$
we have:
$$sum_{n=1}^{+infty}frac{zeta(2n)}{2^{2n-1}} = int_{0}^{+infty}frac{sinh(x/2)}{e^x-1},dx =frac{1}{2}int_{0}^{+infty}e^{-x/2},dx = color{red}{1}.$$






share|cite|improve this answer

















  • 2




    @mathworker21: I can mention that the dominated (or just monotone) convergence theorem clearly applies here.
    – Jack D'Aurizio
    Jul 14 '17 at 18:41





















107














$$
begin{align}
sum_{n=1}^inftyfrac{zeta(2n)}{2^{2n-1}}
&=sum_{n=1}^inftysum_{k=1}^inftyfrac2{k^{2n}2^{2n}}tag{1}\
&=2sum_{k=1}^inftysum_{n=1}^inftyfrac1{(4k^2)^n}tag{2}\
&=2sum_{k=1}^inftyfrac1{4k^2-1}tag{3}\
&=sum_{k=1}^inftyleft(frac1{2k-1}-frac1{2k+1}right)tag{4}\[6pt]
&=1tag{5}
end{align}
$$
Explanation:

$(1)$: expand $zeta(2n)=sumlimits_{k=1}^inftyfrac1{k^{2n}}$

$(2)$: change the order of summation

$(3)$: sum of a geometric series

$(4)$: partial fractions

$(5)$: telescoping sum






share|cite|improve this answer























  • @robjohn, in $(3)$ the numerator should have a $4k^2$. $$sum_{n=1}^{infty} frac{1}{(4k^2)^n} = frac{1}{1- frac{1}{4k^2}} = frac{4k^2}{4k^2 - 1}$$
    – Amad27
    Mar 20 '15 at 16:45








  • 2




    @Amad27: Notice that the sum starts from $n=1$. This means that the sum is $dfrac{frac1{4k^2}}{1-frac1{4k^2}}$.
    – robjohn
    Mar 20 '15 at 17:10












  • Ah! I couldn't catch the index! Sorry robjohn!
    – Amad27
    Mar 20 '15 at 18:17






  • 1




    This one appears to be simplest possible answer. +1 It is as if you see the result in question at the level of $1 + r + r^{2} + cdots = 1/(1 - r)$.
    – Paramanand Singh
    Jun 20 '16 at 10:29



















37














The Laurent expansion of $cot (z)$ at the origin in terms of the Riemann zeta function is $$ cot (z) = frac{1}{z} - 2 sum_{k=1}^{infty}zeta(2k) frac{z^{2k-1}}{pi^{2k}} , 0 < |z| < pi. $$



Letting $ displaystyle z= frac{pi}{2}$, $$cot left(frac{pi}{2} right) = frac{2}{pi} - frac{2}{pi} sum_{k=1}^{infty} frac{zeta(2k)}{2^{2k-1}}.$$



But $cot left(frac{pi}{2} right)=0$.



Therefore,



$$ sum_{k=1}^{infty} frac{zeta(2k)}{2^{2k-1}} = 1.$$






share|cite|improve this answer























  • +1. Nice. I was doing something like that with identities $color{black}{bf 6.3.14}$ and $color{black}{bf 6.3.15}$ but I left because it was cumbersome.
    – Felix Marin
    Dec 24 '14 at 22:54












  • @FelixMarin Thanks. This actually wasn't my first approach.
    – Random Variable
    Dec 24 '14 at 23:20












  • (+1) Beautiful approach! This relation is also proven in this answer.
    – robjohn
    Dec 25 '14 at 0:06



















9














$newcommand{angles}[1]{leftlangle{#1}rightrangle}
newcommand{braces}[1]{leftlbrace{#1}rightrbrace}
newcommand{bracks}[1]{leftlbrack{#1}rightrbrack}
newcommand{dd}{mathrm{d}}
newcommand{ds}[1]{displaystyle{#1}}
newcommand{expo}[1]{,mathrm{e}^{#1},}
newcommand{half}{{1 over 2}}
newcommand{ic}{mathrm{i}}
newcommand{iff}{Leftrightarrow}
newcommand{imp}{Longrightarrow}
newcommand{ol}[1]{overline{#1}}
newcommand{pars}[1]{left({#1}right)}
newcommand{partiald}[3]{frac{partial^{#1} #2}{partial #3^{#1}}}
newcommand{root}[2]{,sqrt[#1]{,{#2},},}
newcommand{totald}[3]{frac{mathrm{d}^{#1} #2}{mathrm{d} #3^{#1}}}
newcommand{verts}[1]{leftvert{#1}rightvert}$



begin{align}
&bbox[10px,#ffd]{sum_{n = 1}^{infty}{zetapars{2n} over 2^{2n - 1}}} =
sum_{n = 2}^{infty}{zetapars{n} over 2^{n - 1}} - sum_{n = 1}^{infty}{zetapars{2n + 1} over 2^{2n}}
\[3mm] = &
-sum_{n = 2}^{infty}pars{-1}^{n}zetapars{n}pars{-,half}^{n - 1} -
sum_{n = 1}^{infty}bracks{zetapars{2n + 1} - 1}pars{half}^{2n} -
underbrace{sum_{n = 1}^{infty}pars{half}^{2n}}_{ds{1 over 3}}
\ = &
-bracks{Psipars{1 + z} + gamma}_{ z = -1/2}
\[3mm] & - bracks{%
{1 over 2z} - half,picotpars{pi z} - {1 over 1 - z^{2}} + 1 - gamma - Psipars{1 + z}}_{ z = 1/2} - {1 over 3}
\[8mm] = &
-Psipars{half} - {2 over 3} +
underbrace{Psipars{3 over 2}}_{ds{Psipars{1/2} + 1/pars{1/2}}} -
{1 over 3} = color{#f00}{1}
end{align}




$Psi$ and $gamma$ are the Digamma function and the Euler-Mascheroni constant, respectively. We used the Digamma Recurrence Formula $ds{Psipars{z + 1} = Psipars{z} + 1/z}$ and the identities $mathbf{6.3.14}$ and $mathbf{6.3.15}$ in this link.







share|cite|improve this answer























    Your Answer





    StackExchange.ifUsing("editor", function () {
    return StackExchange.using("mathjaxEditing", function () {
    StackExchange.MarkdownEditor.creationCallbacks.add(function (editor, postfix) {
    StackExchange.mathjaxEditing.prepareWmdForMathJax(editor, postfix, [["$", "$"], ["\\(","\\)"]]);
    });
    });
    }, "mathjax-editing");

    StackExchange.ready(function() {
    var channelOptions = {
    tags: "".split(" "),
    id: "69"
    };
    initTagRenderer("".split(" "), "".split(" "), channelOptions);

    StackExchange.using("externalEditor", function() {
    // Have to fire editor after snippets, if snippets enabled
    if (StackExchange.settings.snippets.snippetsEnabled) {
    StackExchange.using("snippets", function() {
    createEditor();
    });
    }
    else {
    createEditor();
    }
    });

    function createEditor() {
    StackExchange.prepareEditor({
    heartbeatType: 'answer',
    autoActivateHeartbeat: false,
    convertImagesToLinks: true,
    noModals: true,
    showLowRepImageUploadWarning: true,
    reputationToPostImages: 10,
    bindNavPrevention: true,
    postfix: "",
    imageUploader: {
    brandingHtml: "Powered by u003ca class="icon-imgur-white" href="https://imgur.com/"u003eu003c/au003e",
    contentPolicyHtml: "User contributions licensed under u003ca href="https://creativecommons.org/licenses/by-sa/3.0/"u003ecc by-sa 3.0 with attribution requiredu003c/au003e u003ca href="https://stackoverflow.com/legal/content-policy"u003e(content policy)u003c/au003e",
    allowUrls: true
    },
    noCode: true, onDemand: true,
    discardSelector: ".discard-answer"
    ,immediatelyShowMarkdownHelp:true
    });


    }
    });














    draft saved

    draft discarded


















    StackExchange.ready(
    function () {
    StackExchange.openid.initPostLogin('.new-post-login', 'https%3a%2f%2fmath.stackexchange.com%2fquestions%2f1080000%2fhow-to-prove-that-frac-zeta2-2-frac-zeta-423-frac-zeta-62%23new-answer', 'question_page');
    }
    );

    Post as a guest















    Required, but never shown

























    4 Answers
    4






    active

    oldest

    votes








    4 Answers
    4






    active

    oldest

    votes









    active

    oldest

    votes






    active

    oldest

    votes









    107














    Since
    $$zeta(2n) = frac{1}{(2n-1)!}int_{0}^{+infty}frac{x^{2n-1}}{e^x-1},dx $$
    we have:
    $$sum_{n=1}^{+infty}frac{zeta(2n)}{2^{2n-1}} = int_{0}^{+infty}frac{sinh(x/2)}{e^x-1},dx =frac{1}{2}int_{0}^{+infty}e^{-x/2},dx = color{red}{1}.$$






    share|cite|improve this answer

















    • 2




      @mathworker21: I can mention that the dominated (or just monotone) convergence theorem clearly applies here.
      – Jack D'Aurizio
      Jul 14 '17 at 18:41


















    107














    Since
    $$zeta(2n) = frac{1}{(2n-1)!}int_{0}^{+infty}frac{x^{2n-1}}{e^x-1},dx $$
    we have:
    $$sum_{n=1}^{+infty}frac{zeta(2n)}{2^{2n-1}} = int_{0}^{+infty}frac{sinh(x/2)}{e^x-1},dx =frac{1}{2}int_{0}^{+infty}e^{-x/2},dx = color{red}{1}.$$






    share|cite|improve this answer

















    • 2




      @mathworker21: I can mention that the dominated (or just monotone) convergence theorem clearly applies here.
      – Jack D'Aurizio
      Jul 14 '17 at 18:41
















    107












    107








    107






    Since
    $$zeta(2n) = frac{1}{(2n-1)!}int_{0}^{+infty}frac{x^{2n-1}}{e^x-1},dx $$
    we have:
    $$sum_{n=1}^{+infty}frac{zeta(2n)}{2^{2n-1}} = int_{0}^{+infty}frac{sinh(x/2)}{e^x-1},dx =frac{1}{2}int_{0}^{+infty}e^{-x/2},dx = color{red}{1}.$$






    share|cite|improve this answer












    Since
    $$zeta(2n) = frac{1}{(2n-1)!}int_{0}^{+infty}frac{x^{2n-1}}{e^x-1},dx $$
    we have:
    $$sum_{n=1}^{+infty}frac{zeta(2n)}{2^{2n-1}} = int_{0}^{+infty}frac{sinh(x/2)}{e^x-1},dx =frac{1}{2}int_{0}^{+infty}e^{-x/2},dx = color{red}{1}.$$







    share|cite|improve this answer












    share|cite|improve this answer



    share|cite|improve this answer










    answered Dec 24 '14 at 18:29









    Jack D'Aurizio

    286k33279656




    286k33279656








    • 2




      @mathworker21: I can mention that the dominated (or just monotone) convergence theorem clearly applies here.
      – Jack D'Aurizio
      Jul 14 '17 at 18:41
















    • 2




      @mathworker21: I can mention that the dominated (or just monotone) convergence theorem clearly applies here.
      – Jack D'Aurizio
      Jul 14 '17 at 18:41










    2




    2




    @mathworker21: I can mention that the dominated (or just monotone) convergence theorem clearly applies here.
    – Jack D'Aurizio
    Jul 14 '17 at 18:41






    @mathworker21: I can mention that the dominated (or just monotone) convergence theorem clearly applies here.
    – Jack D'Aurizio
    Jul 14 '17 at 18:41













    107














    $$
    begin{align}
    sum_{n=1}^inftyfrac{zeta(2n)}{2^{2n-1}}
    &=sum_{n=1}^inftysum_{k=1}^inftyfrac2{k^{2n}2^{2n}}tag{1}\
    &=2sum_{k=1}^inftysum_{n=1}^inftyfrac1{(4k^2)^n}tag{2}\
    &=2sum_{k=1}^inftyfrac1{4k^2-1}tag{3}\
    &=sum_{k=1}^inftyleft(frac1{2k-1}-frac1{2k+1}right)tag{4}\[6pt]
    &=1tag{5}
    end{align}
    $$
    Explanation:

    $(1)$: expand $zeta(2n)=sumlimits_{k=1}^inftyfrac1{k^{2n}}$

    $(2)$: change the order of summation

    $(3)$: sum of a geometric series

    $(4)$: partial fractions

    $(5)$: telescoping sum






    share|cite|improve this answer























    • @robjohn, in $(3)$ the numerator should have a $4k^2$. $$sum_{n=1}^{infty} frac{1}{(4k^2)^n} = frac{1}{1- frac{1}{4k^2}} = frac{4k^2}{4k^2 - 1}$$
      – Amad27
      Mar 20 '15 at 16:45








    • 2




      @Amad27: Notice that the sum starts from $n=1$. This means that the sum is $dfrac{frac1{4k^2}}{1-frac1{4k^2}}$.
      – robjohn
      Mar 20 '15 at 17:10












    • Ah! I couldn't catch the index! Sorry robjohn!
      – Amad27
      Mar 20 '15 at 18:17






    • 1




      This one appears to be simplest possible answer. +1 It is as if you see the result in question at the level of $1 + r + r^{2} + cdots = 1/(1 - r)$.
      – Paramanand Singh
      Jun 20 '16 at 10:29
















    107














    $$
    begin{align}
    sum_{n=1}^inftyfrac{zeta(2n)}{2^{2n-1}}
    &=sum_{n=1}^inftysum_{k=1}^inftyfrac2{k^{2n}2^{2n}}tag{1}\
    &=2sum_{k=1}^inftysum_{n=1}^inftyfrac1{(4k^2)^n}tag{2}\
    &=2sum_{k=1}^inftyfrac1{4k^2-1}tag{3}\
    &=sum_{k=1}^inftyleft(frac1{2k-1}-frac1{2k+1}right)tag{4}\[6pt]
    &=1tag{5}
    end{align}
    $$
    Explanation:

    $(1)$: expand $zeta(2n)=sumlimits_{k=1}^inftyfrac1{k^{2n}}$

    $(2)$: change the order of summation

    $(3)$: sum of a geometric series

    $(4)$: partial fractions

    $(5)$: telescoping sum






    share|cite|improve this answer























    • @robjohn, in $(3)$ the numerator should have a $4k^2$. $$sum_{n=1}^{infty} frac{1}{(4k^2)^n} = frac{1}{1- frac{1}{4k^2}} = frac{4k^2}{4k^2 - 1}$$
      – Amad27
      Mar 20 '15 at 16:45








    • 2




      @Amad27: Notice that the sum starts from $n=1$. This means that the sum is $dfrac{frac1{4k^2}}{1-frac1{4k^2}}$.
      – robjohn
      Mar 20 '15 at 17:10












    • Ah! I couldn't catch the index! Sorry robjohn!
      – Amad27
      Mar 20 '15 at 18:17






    • 1




      This one appears to be simplest possible answer. +1 It is as if you see the result in question at the level of $1 + r + r^{2} + cdots = 1/(1 - r)$.
      – Paramanand Singh
      Jun 20 '16 at 10:29














    107












    107








    107






    $$
    begin{align}
    sum_{n=1}^inftyfrac{zeta(2n)}{2^{2n-1}}
    &=sum_{n=1}^inftysum_{k=1}^inftyfrac2{k^{2n}2^{2n}}tag{1}\
    &=2sum_{k=1}^inftysum_{n=1}^inftyfrac1{(4k^2)^n}tag{2}\
    &=2sum_{k=1}^inftyfrac1{4k^2-1}tag{3}\
    &=sum_{k=1}^inftyleft(frac1{2k-1}-frac1{2k+1}right)tag{4}\[6pt]
    &=1tag{5}
    end{align}
    $$
    Explanation:

    $(1)$: expand $zeta(2n)=sumlimits_{k=1}^inftyfrac1{k^{2n}}$

    $(2)$: change the order of summation

    $(3)$: sum of a geometric series

    $(4)$: partial fractions

    $(5)$: telescoping sum






    share|cite|improve this answer














    $$
    begin{align}
    sum_{n=1}^inftyfrac{zeta(2n)}{2^{2n-1}}
    &=sum_{n=1}^inftysum_{k=1}^inftyfrac2{k^{2n}2^{2n}}tag{1}\
    &=2sum_{k=1}^inftysum_{n=1}^inftyfrac1{(4k^2)^n}tag{2}\
    &=2sum_{k=1}^inftyfrac1{4k^2-1}tag{3}\
    &=sum_{k=1}^inftyleft(frac1{2k-1}-frac1{2k+1}right)tag{4}\[6pt]
    &=1tag{5}
    end{align}
    $$
    Explanation:

    $(1)$: expand $zeta(2n)=sumlimits_{k=1}^inftyfrac1{k^{2n}}$

    $(2)$: change the order of summation

    $(3)$: sum of a geometric series

    $(4)$: partial fractions

    $(5)$: telescoping sum







    share|cite|improve this answer














    share|cite|improve this answer



    share|cite|improve this answer








    edited Dec 24 '14 at 20:49

























    answered Dec 24 '14 at 20:44









    robjohn

    264k27303623




    264k27303623












    • @robjohn, in $(3)$ the numerator should have a $4k^2$. $$sum_{n=1}^{infty} frac{1}{(4k^2)^n} = frac{1}{1- frac{1}{4k^2}} = frac{4k^2}{4k^2 - 1}$$
      – Amad27
      Mar 20 '15 at 16:45








    • 2




      @Amad27: Notice that the sum starts from $n=1$. This means that the sum is $dfrac{frac1{4k^2}}{1-frac1{4k^2}}$.
      – robjohn
      Mar 20 '15 at 17:10












    • Ah! I couldn't catch the index! Sorry robjohn!
      – Amad27
      Mar 20 '15 at 18:17






    • 1




      This one appears to be simplest possible answer. +1 It is as if you see the result in question at the level of $1 + r + r^{2} + cdots = 1/(1 - r)$.
      – Paramanand Singh
      Jun 20 '16 at 10:29


















    • @robjohn, in $(3)$ the numerator should have a $4k^2$. $$sum_{n=1}^{infty} frac{1}{(4k^2)^n} = frac{1}{1- frac{1}{4k^2}} = frac{4k^2}{4k^2 - 1}$$
      – Amad27
      Mar 20 '15 at 16:45








    • 2




      @Amad27: Notice that the sum starts from $n=1$. This means that the sum is $dfrac{frac1{4k^2}}{1-frac1{4k^2}}$.
      – robjohn
      Mar 20 '15 at 17:10












    • Ah! I couldn't catch the index! Sorry robjohn!
      – Amad27
      Mar 20 '15 at 18:17






    • 1




      This one appears to be simplest possible answer. +1 It is as if you see the result in question at the level of $1 + r + r^{2} + cdots = 1/(1 - r)$.
      – Paramanand Singh
      Jun 20 '16 at 10:29
















    @robjohn, in $(3)$ the numerator should have a $4k^2$. $$sum_{n=1}^{infty} frac{1}{(4k^2)^n} = frac{1}{1- frac{1}{4k^2}} = frac{4k^2}{4k^2 - 1}$$
    – Amad27
    Mar 20 '15 at 16:45






    @robjohn, in $(3)$ the numerator should have a $4k^2$. $$sum_{n=1}^{infty} frac{1}{(4k^2)^n} = frac{1}{1- frac{1}{4k^2}} = frac{4k^2}{4k^2 - 1}$$
    – Amad27
    Mar 20 '15 at 16:45






    2




    2




    @Amad27: Notice that the sum starts from $n=1$. This means that the sum is $dfrac{frac1{4k^2}}{1-frac1{4k^2}}$.
    – robjohn
    Mar 20 '15 at 17:10






    @Amad27: Notice that the sum starts from $n=1$. This means that the sum is $dfrac{frac1{4k^2}}{1-frac1{4k^2}}$.
    – robjohn
    Mar 20 '15 at 17:10














    Ah! I couldn't catch the index! Sorry robjohn!
    – Amad27
    Mar 20 '15 at 18:17




    Ah! I couldn't catch the index! Sorry robjohn!
    – Amad27
    Mar 20 '15 at 18:17




    1




    1




    This one appears to be simplest possible answer. +1 It is as if you see the result in question at the level of $1 + r + r^{2} + cdots = 1/(1 - r)$.
    – Paramanand Singh
    Jun 20 '16 at 10:29




    This one appears to be simplest possible answer. +1 It is as if you see the result in question at the level of $1 + r + r^{2} + cdots = 1/(1 - r)$.
    – Paramanand Singh
    Jun 20 '16 at 10:29











    37














    The Laurent expansion of $cot (z)$ at the origin in terms of the Riemann zeta function is $$ cot (z) = frac{1}{z} - 2 sum_{k=1}^{infty}zeta(2k) frac{z^{2k-1}}{pi^{2k}} , 0 < |z| < pi. $$



    Letting $ displaystyle z= frac{pi}{2}$, $$cot left(frac{pi}{2} right) = frac{2}{pi} - frac{2}{pi} sum_{k=1}^{infty} frac{zeta(2k)}{2^{2k-1}}.$$



    But $cot left(frac{pi}{2} right)=0$.



    Therefore,



    $$ sum_{k=1}^{infty} frac{zeta(2k)}{2^{2k-1}} = 1.$$






    share|cite|improve this answer























    • +1. Nice. I was doing something like that with identities $color{black}{bf 6.3.14}$ and $color{black}{bf 6.3.15}$ but I left because it was cumbersome.
      – Felix Marin
      Dec 24 '14 at 22:54












    • @FelixMarin Thanks. This actually wasn't my first approach.
      – Random Variable
      Dec 24 '14 at 23:20












    • (+1) Beautiful approach! This relation is also proven in this answer.
      – robjohn
      Dec 25 '14 at 0:06
















    37














    The Laurent expansion of $cot (z)$ at the origin in terms of the Riemann zeta function is $$ cot (z) = frac{1}{z} - 2 sum_{k=1}^{infty}zeta(2k) frac{z^{2k-1}}{pi^{2k}} , 0 < |z| < pi. $$



    Letting $ displaystyle z= frac{pi}{2}$, $$cot left(frac{pi}{2} right) = frac{2}{pi} - frac{2}{pi} sum_{k=1}^{infty} frac{zeta(2k)}{2^{2k-1}}.$$



    But $cot left(frac{pi}{2} right)=0$.



    Therefore,



    $$ sum_{k=1}^{infty} frac{zeta(2k)}{2^{2k-1}} = 1.$$






    share|cite|improve this answer























    • +1. Nice. I was doing something like that with identities $color{black}{bf 6.3.14}$ and $color{black}{bf 6.3.15}$ but I left because it was cumbersome.
      – Felix Marin
      Dec 24 '14 at 22:54












    • @FelixMarin Thanks. This actually wasn't my first approach.
      – Random Variable
      Dec 24 '14 at 23:20












    • (+1) Beautiful approach! This relation is also proven in this answer.
      – robjohn
      Dec 25 '14 at 0:06














    37












    37








    37






    The Laurent expansion of $cot (z)$ at the origin in terms of the Riemann zeta function is $$ cot (z) = frac{1}{z} - 2 sum_{k=1}^{infty}zeta(2k) frac{z^{2k-1}}{pi^{2k}} , 0 < |z| < pi. $$



    Letting $ displaystyle z= frac{pi}{2}$, $$cot left(frac{pi}{2} right) = frac{2}{pi} - frac{2}{pi} sum_{k=1}^{infty} frac{zeta(2k)}{2^{2k-1}}.$$



    But $cot left(frac{pi}{2} right)=0$.



    Therefore,



    $$ sum_{k=1}^{infty} frac{zeta(2k)}{2^{2k-1}} = 1.$$






    share|cite|improve this answer














    The Laurent expansion of $cot (z)$ at the origin in terms of the Riemann zeta function is $$ cot (z) = frac{1}{z} - 2 sum_{k=1}^{infty}zeta(2k) frac{z^{2k-1}}{pi^{2k}} , 0 < |z| < pi. $$



    Letting $ displaystyle z= frac{pi}{2}$, $$cot left(frac{pi}{2} right) = frac{2}{pi} - frac{2}{pi} sum_{k=1}^{infty} frac{zeta(2k)}{2^{2k-1}}.$$



    But $cot left(frac{pi}{2} right)=0$.



    Therefore,



    $$ sum_{k=1}^{infty} frac{zeta(2k)}{2^{2k-1}} = 1.$$







    share|cite|improve this answer














    share|cite|improve this answer



    share|cite|improve this answer








    edited Dec 24 '14 at 22:33

























    answered Dec 24 '14 at 22:27









    Random Variable

    25.4k170136




    25.4k170136












    • +1. Nice. I was doing something like that with identities $color{black}{bf 6.3.14}$ and $color{black}{bf 6.3.15}$ but I left because it was cumbersome.
      – Felix Marin
      Dec 24 '14 at 22:54












    • @FelixMarin Thanks. This actually wasn't my first approach.
      – Random Variable
      Dec 24 '14 at 23:20












    • (+1) Beautiful approach! This relation is also proven in this answer.
      – robjohn
      Dec 25 '14 at 0:06


















    • +1. Nice. I was doing something like that with identities $color{black}{bf 6.3.14}$ and $color{black}{bf 6.3.15}$ but I left because it was cumbersome.
      – Felix Marin
      Dec 24 '14 at 22:54












    • @FelixMarin Thanks. This actually wasn't my first approach.
      – Random Variable
      Dec 24 '14 at 23:20












    • (+1) Beautiful approach! This relation is also proven in this answer.
      – robjohn
      Dec 25 '14 at 0:06
















    +1. Nice. I was doing something like that with identities $color{black}{bf 6.3.14}$ and $color{black}{bf 6.3.15}$ but I left because it was cumbersome.
    – Felix Marin
    Dec 24 '14 at 22:54






    +1. Nice. I was doing something like that with identities $color{black}{bf 6.3.14}$ and $color{black}{bf 6.3.15}$ but I left because it was cumbersome.
    – Felix Marin
    Dec 24 '14 at 22:54














    @FelixMarin Thanks. This actually wasn't my first approach.
    – Random Variable
    Dec 24 '14 at 23:20






    @FelixMarin Thanks. This actually wasn't my first approach.
    – Random Variable
    Dec 24 '14 at 23:20














    (+1) Beautiful approach! This relation is also proven in this answer.
    – robjohn
    Dec 25 '14 at 0:06




    (+1) Beautiful approach! This relation is also proven in this answer.
    – robjohn
    Dec 25 '14 at 0:06











    9














    $newcommand{angles}[1]{leftlangle{#1}rightrangle}
    newcommand{braces}[1]{leftlbrace{#1}rightrbrace}
    newcommand{bracks}[1]{leftlbrack{#1}rightrbrack}
    newcommand{dd}{mathrm{d}}
    newcommand{ds}[1]{displaystyle{#1}}
    newcommand{expo}[1]{,mathrm{e}^{#1},}
    newcommand{half}{{1 over 2}}
    newcommand{ic}{mathrm{i}}
    newcommand{iff}{Leftrightarrow}
    newcommand{imp}{Longrightarrow}
    newcommand{ol}[1]{overline{#1}}
    newcommand{pars}[1]{left({#1}right)}
    newcommand{partiald}[3]{frac{partial^{#1} #2}{partial #3^{#1}}}
    newcommand{root}[2]{,sqrt[#1]{,{#2},},}
    newcommand{totald}[3]{frac{mathrm{d}^{#1} #2}{mathrm{d} #3^{#1}}}
    newcommand{verts}[1]{leftvert{#1}rightvert}$



    begin{align}
    &bbox[10px,#ffd]{sum_{n = 1}^{infty}{zetapars{2n} over 2^{2n - 1}}} =
    sum_{n = 2}^{infty}{zetapars{n} over 2^{n - 1}} - sum_{n = 1}^{infty}{zetapars{2n + 1} over 2^{2n}}
    \[3mm] = &
    -sum_{n = 2}^{infty}pars{-1}^{n}zetapars{n}pars{-,half}^{n - 1} -
    sum_{n = 1}^{infty}bracks{zetapars{2n + 1} - 1}pars{half}^{2n} -
    underbrace{sum_{n = 1}^{infty}pars{half}^{2n}}_{ds{1 over 3}}
    \ = &
    -bracks{Psipars{1 + z} + gamma}_{ z = -1/2}
    \[3mm] & - bracks{%
    {1 over 2z} - half,picotpars{pi z} - {1 over 1 - z^{2}} + 1 - gamma - Psipars{1 + z}}_{ z = 1/2} - {1 over 3}
    \[8mm] = &
    -Psipars{half} - {2 over 3} +
    underbrace{Psipars{3 over 2}}_{ds{Psipars{1/2} + 1/pars{1/2}}} -
    {1 over 3} = color{#f00}{1}
    end{align}




    $Psi$ and $gamma$ are the Digamma function and the Euler-Mascheroni constant, respectively. We used the Digamma Recurrence Formula $ds{Psipars{z + 1} = Psipars{z} + 1/z}$ and the identities $mathbf{6.3.14}$ and $mathbf{6.3.15}$ in this link.







    share|cite|improve this answer




























      9














      $newcommand{angles}[1]{leftlangle{#1}rightrangle}
      newcommand{braces}[1]{leftlbrace{#1}rightrbrace}
      newcommand{bracks}[1]{leftlbrack{#1}rightrbrack}
      newcommand{dd}{mathrm{d}}
      newcommand{ds}[1]{displaystyle{#1}}
      newcommand{expo}[1]{,mathrm{e}^{#1},}
      newcommand{half}{{1 over 2}}
      newcommand{ic}{mathrm{i}}
      newcommand{iff}{Leftrightarrow}
      newcommand{imp}{Longrightarrow}
      newcommand{ol}[1]{overline{#1}}
      newcommand{pars}[1]{left({#1}right)}
      newcommand{partiald}[3]{frac{partial^{#1} #2}{partial #3^{#1}}}
      newcommand{root}[2]{,sqrt[#1]{,{#2},},}
      newcommand{totald}[3]{frac{mathrm{d}^{#1} #2}{mathrm{d} #3^{#1}}}
      newcommand{verts}[1]{leftvert{#1}rightvert}$



      begin{align}
      &bbox[10px,#ffd]{sum_{n = 1}^{infty}{zetapars{2n} over 2^{2n - 1}}} =
      sum_{n = 2}^{infty}{zetapars{n} over 2^{n - 1}} - sum_{n = 1}^{infty}{zetapars{2n + 1} over 2^{2n}}
      \[3mm] = &
      -sum_{n = 2}^{infty}pars{-1}^{n}zetapars{n}pars{-,half}^{n - 1} -
      sum_{n = 1}^{infty}bracks{zetapars{2n + 1} - 1}pars{half}^{2n} -
      underbrace{sum_{n = 1}^{infty}pars{half}^{2n}}_{ds{1 over 3}}
      \ = &
      -bracks{Psipars{1 + z} + gamma}_{ z = -1/2}
      \[3mm] & - bracks{%
      {1 over 2z} - half,picotpars{pi z} - {1 over 1 - z^{2}} + 1 - gamma - Psipars{1 + z}}_{ z = 1/2} - {1 over 3}
      \[8mm] = &
      -Psipars{half} - {2 over 3} +
      underbrace{Psipars{3 over 2}}_{ds{Psipars{1/2} + 1/pars{1/2}}} -
      {1 over 3} = color{#f00}{1}
      end{align}




      $Psi$ and $gamma$ are the Digamma function and the Euler-Mascheroni constant, respectively. We used the Digamma Recurrence Formula $ds{Psipars{z + 1} = Psipars{z} + 1/z}$ and the identities $mathbf{6.3.14}$ and $mathbf{6.3.15}$ in this link.







      share|cite|improve this answer


























        9












        9








        9






        $newcommand{angles}[1]{leftlangle{#1}rightrangle}
        newcommand{braces}[1]{leftlbrace{#1}rightrbrace}
        newcommand{bracks}[1]{leftlbrack{#1}rightrbrack}
        newcommand{dd}{mathrm{d}}
        newcommand{ds}[1]{displaystyle{#1}}
        newcommand{expo}[1]{,mathrm{e}^{#1},}
        newcommand{half}{{1 over 2}}
        newcommand{ic}{mathrm{i}}
        newcommand{iff}{Leftrightarrow}
        newcommand{imp}{Longrightarrow}
        newcommand{ol}[1]{overline{#1}}
        newcommand{pars}[1]{left({#1}right)}
        newcommand{partiald}[3]{frac{partial^{#1} #2}{partial #3^{#1}}}
        newcommand{root}[2]{,sqrt[#1]{,{#2},},}
        newcommand{totald}[3]{frac{mathrm{d}^{#1} #2}{mathrm{d} #3^{#1}}}
        newcommand{verts}[1]{leftvert{#1}rightvert}$



        begin{align}
        &bbox[10px,#ffd]{sum_{n = 1}^{infty}{zetapars{2n} over 2^{2n - 1}}} =
        sum_{n = 2}^{infty}{zetapars{n} over 2^{n - 1}} - sum_{n = 1}^{infty}{zetapars{2n + 1} over 2^{2n}}
        \[3mm] = &
        -sum_{n = 2}^{infty}pars{-1}^{n}zetapars{n}pars{-,half}^{n - 1} -
        sum_{n = 1}^{infty}bracks{zetapars{2n + 1} - 1}pars{half}^{2n} -
        underbrace{sum_{n = 1}^{infty}pars{half}^{2n}}_{ds{1 over 3}}
        \ = &
        -bracks{Psipars{1 + z} + gamma}_{ z = -1/2}
        \[3mm] & - bracks{%
        {1 over 2z} - half,picotpars{pi z} - {1 over 1 - z^{2}} + 1 - gamma - Psipars{1 + z}}_{ z = 1/2} - {1 over 3}
        \[8mm] = &
        -Psipars{half} - {2 over 3} +
        underbrace{Psipars{3 over 2}}_{ds{Psipars{1/2} + 1/pars{1/2}}} -
        {1 over 3} = color{#f00}{1}
        end{align}




        $Psi$ and $gamma$ are the Digamma function and the Euler-Mascheroni constant, respectively. We used the Digamma Recurrence Formula $ds{Psipars{z + 1} = Psipars{z} + 1/z}$ and the identities $mathbf{6.3.14}$ and $mathbf{6.3.15}$ in this link.







        share|cite|improve this answer














        $newcommand{angles}[1]{leftlangle{#1}rightrangle}
        newcommand{braces}[1]{leftlbrace{#1}rightrbrace}
        newcommand{bracks}[1]{leftlbrack{#1}rightrbrack}
        newcommand{dd}{mathrm{d}}
        newcommand{ds}[1]{displaystyle{#1}}
        newcommand{expo}[1]{,mathrm{e}^{#1},}
        newcommand{half}{{1 over 2}}
        newcommand{ic}{mathrm{i}}
        newcommand{iff}{Leftrightarrow}
        newcommand{imp}{Longrightarrow}
        newcommand{ol}[1]{overline{#1}}
        newcommand{pars}[1]{left({#1}right)}
        newcommand{partiald}[3]{frac{partial^{#1} #2}{partial #3^{#1}}}
        newcommand{root}[2]{,sqrt[#1]{,{#2},},}
        newcommand{totald}[3]{frac{mathrm{d}^{#1} #2}{mathrm{d} #3^{#1}}}
        newcommand{verts}[1]{leftvert{#1}rightvert}$



        begin{align}
        &bbox[10px,#ffd]{sum_{n = 1}^{infty}{zetapars{2n} over 2^{2n - 1}}} =
        sum_{n = 2}^{infty}{zetapars{n} over 2^{n - 1}} - sum_{n = 1}^{infty}{zetapars{2n + 1} over 2^{2n}}
        \[3mm] = &
        -sum_{n = 2}^{infty}pars{-1}^{n}zetapars{n}pars{-,half}^{n - 1} -
        sum_{n = 1}^{infty}bracks{zetapars{2n + 1} - 1}pars{half}^{2n} -
        underbrace{sum_{n = 1}^{infty}pars{half}^{2n}}_{ds{1 over 3}}
        \ = &
        -bracks{Psipars{1 + z} + gamma}_{ z = -1/2}
        \[3mm] & - bracks{%
        {1 over 2z} - half,picotpars{pi z} - {1 over 1 - z^{2}} + 1 - gamma - Psipars{1 + z}}_{ z = 1/2} - {1 over 3}
        \[8mm] = &
        -Psipars{half} - {2 over 3} +
        underbrace{Psipars{3 over 2}}_{ds{Psipars{1/2} + 1/pars{1/2}}} -
        {1 over 3} = color{#f00}{1}
        end{align}




        $Psi$ and $gamma$ are the Digamma function and the Euler-Mascheroni constant, respectively. We used the Digamma Recurrence Formula $ds{Psipars{z + 1} = Psipars{z} + 1/z}$ and the identities $mathbf{6.3.14}$ and $mathbf{6.3.15}$ in this link.








        share|cite|improve this answer














        share|cite|improve this answer



        share|cite|improve this answer








        edited Nov 19 '18 at 22:52

























        answered Jun 19 '16 at 19:42









        Felix Marin

        67.1k7107141




        67.1k7107141






























            draft saved

            draft discarded




















































            Thanks for contributing an answer to Mathematics Stack Exchange!


            • Please be sure to answer the question. Provide details and share your research!

            But avoid



            • Asking for help, clarification, or responding to other answers.

            • Making statements based on opinion; back them up with references or personal experience.


            Use MathJax to format equations. MathJax reference.


            To learn more, see our tips on writing great answers.





            Some of your past answers have not been well-received, and you're in danger of being blocked from answering.


            Please pay close attention to the following guidance:


            • Please be sure to answer the question. Provide details and share your research!

            But avoid



            • Asking for help, clarification, or responding to other answers.

            • Making statements based on opinion; back them up with references or personal experience.


            To learn more, see our tips on writing great answers.




            draft saved


            draft discarded














            StackExchange.ready(
            function () {
            StackExchange.openid.initPostLogin('.new-post-login', 'https%3a%2f%2fmath.stackexchange.com%2fquestions%2f1080000%2fhow-to-prove-that-frac-zeta2-2-frac-zeta-423-frac-zeta-62%23new-answer', 'question_page');
            }
            );

            Post as a guest















            Required, but never shown





















































            Required, but never shown














            Required, but never shown












            Required, but never shown







            Required, but never shown

































            Required, but never shown














            Required, but never shown












            Required, but never shown







            Required, but never shown







            Popular posts from this blog

            Can a sorcerer learn a 5th-level spell early by creating spell slots using the Font of Magic feature?

            Does disintegrating a polymorphed enemy still kill it after the 2018 errata?

            A Topological Invariant for $pi_3(U(n))$